Use \dd for derivative d in Y&F-12 28.12 and 30
[course.git] / latex / problems / Serway_and_Jewett_4_venkat / problem12.V3.tex
1 \begin{problem}
2 A force of $64\U{N}$ stretches a certain spring by $16\U{cm}$. A block
3 of mass $4\U{kg}$ is hung from the spring and the spring-mass
4 combination executes vertical simple harmonic oscillations when at
5 $t=0$, the mass is pulled down by $12\U{cm}$ below its equilibrium
6 position and released from rest. Assume gravitational potential energy
7 is zero at the highest position ($y=0$) of oscillation of the mass
8 $m$. \\
9 \Part{a} Write down an equation for position of the mass assuming the
10 clock reads $t=0$ at the instant the mass is released (i.e~at the
11 lowest point of vertical oscillations). \\
12 \Part{b} Find the shortest time needed for the mass to go through a
13 point $8\U{cm}$ above the equilibrium position from the instant it is
14 released. \\
15 \Part{c} Find the position and the magnitude and direction of its
16 velocity of motion at $t=0.10\pi\U{s}$. \\
17 \Part{d} What is the total energy of the spring mass combination at
18 any instant of oscillation?
19 \end{problem}
20
21 \begin{solution}
22 \Part{a}
23 From the initial condition, we know that the equation must look like
24 \begin{equation}
25   y(t) = -A[\cos(\omega t) - 1] \;,
26 \end{equation}
27 where the $-1$ shifts the $\cos$ down so $y(t)_\text{max}=0$.
28 $A=12\U{cm}$ is given in the problem, but we need to find $\omega$
29 \begin{align}
30   k &= \frac{F}{\Delta l} = 400\U{N/m} \\
31   \omega &= \sqrt{\frac{k}{m}} = \sqrt{\frac{F}{m\Delta l}}
32     = 10\U{rad/s} \\
33   y(t) &= \ans{-12\U{cm}\cdot[\cos(10\U{rad/s}\cdot t)-1]}
34 \end{align}
35
36 \Part{b}
37 Let $\Delta y=8\U{cm}$.  Then
38 \begin{align}
39   y(t)+A &= -A\cos(\omega t) = \Delta y \\
40   \omega t &= \arccos\p({\frac{-\Delta y}{A}}) \\
41   t &= \frac{\arccos\p({\frac{-\Delta y}{A}})}{\omega}
42     = \ans{0.230\U{s}}
43 \end{align}
44
45 \Part{c}
46 We get the position by plugging in
47 \begin{equation}
48   y(t=0.10\pi\U{s}) = -12\U{cm}\cdot[\cos(10\U{rad/s}\cdot 0.10\pi\U{s}) + 1]
49     = -12\U{cm}\cdot[\cos(\pi\U{rad}) + 1]
50     = -12\U{cm}\cdot[-1 + 1]
51     = \ans{0}
52 \end{equation}
53 This is the high point of the oscillation so $v=\ans{0}$
54
55 \Part{d}
56 Energy is conserved, so we can pick our favorite point.  How about
57 when all the energy is stored in the spring.  We'll need to know
58 the total amount the spring stretched though.  At equilibrium
59 \begin{align}
60   mg &= k\Delta L_\text{eq} \\
61   \Delta L_\text{eq} &= \frac{mg}{k} = 9.8\U{cm} \;,
62 \end{align}
63 so
64 \begin{equation}
65   E = \frac{1}{2} k (\Delta L_\text{eq} - A)^2
66     = \frac{1}{2}\cdot 400\U{N/m}\cdot(0.098\U{m} - 0.120\U{m})^2
67     = \ans{96.8\U{mJ}}
68 \end{equation}
69
70 Note: I used $g = 9.8\U{m/s$^2$}$ throughout, while Prof.~Venkat used
71 $10\U{m/s$^2$}$, which is why some of our answers are slightly different.
72 \end{solution}